m-saghaei

New Member
ارسال ها
338
لایک ها
258
امتیاز
0
پاسخ : ماراتن نظریه ی اعداد (سطح ممتاز)

ببخشید کجای راه حل شما شبیه من بود ؟؟؟
همون نامساوی آخریه.سوال بعد رو اگه میشه خودتون لطف کنید.
 

AHZolfaghari

Well-Known Member
ارسال ها
935
لایک ها
1,654
امتیاز
93
پاسخ : ماراتن نظریه ی اعداد (سطح ممتاز)

خب سوال بعد :
اعداد طبیعی a,b طوری هستند که 16a- 15b , 15a + 16b هر دو مربع کامل اند . کمترین مقداری را که مقدار کوچک تر بین این دو مربع کامل می توانند به خود بگیرد را بدست آورید .
 

math1998

New Member
ارسال ها
336
لایک ها
224
امتیاز
0

AHZolfaghari

Well-Known Member
ارسال ها
935
لایک ها
1,654
امتیاز
93

math1998

New Member
ارسال ها
336
لایک ها
224
امتیاز
0
پاسخ : ماراتن نظریه ی اعداد (سطح ممتاز)

تمام
هایی را بیابید که


 

Dadgarnia

New Member
ارسال ها
1,350
لایک ها
1,127
امتیاز
0
پاسخ : ماراتن نظریه ی اعداد (سطح ممتاز)

تمام
هایی را بیابید که


فرض مي كنيم
كه در اون
عددي صحيح و نامنفي و
است. داريم:


حالا اگر
برابر با صفر باشد به راحتي داريم m=1. حالا فرض مي كنيم
بزرگتر از صفر باشد و
پس بدست مي آيد:
براي اثبات رابطه ي آخر هم كافي است داشته باشيم:
كه به راحتي مي توانيم ببينيم براي
صحيح است. با بررسي حالات باقيمانده به جواب هاي ١ و ٢ و ٤ مي رسيم.

---- دو نوشته به هم متصل شده است ----

سوال بعد:
يك چند جمله اي با ضرايب صحيح است و براي هر
طبيعي
است. در دنباله ي
داريم
و براي هر
طبيعي يكي از
ها بر
بخشپذيرند. تمام چند جمله اي هاي
را بيابيد.
 

Dadgarnia

New Member
ارسال ها
1,350
لایک ها
1,127
امتیاز
0
پاسخ : ماراتن نظریه ی اعداد (سطح ممتاز)

برای سوال قبل ثابت کنید تنها چند جمله ای
هست.
سوال بعد:
تمام اعداد صحیح و نامنفی
و عدد اول
را بیابید به طوریکه:

فقط دوستانی که سوال میذارن خیلی سطح سوال رو بالا نبرن و سعی کنین کم کم سوالا رو سخت کنین تا ماراتن دوباره نخوابه!
 

m-saghaei

New Member
ارسال ها
338
لایک ها
258
امتیاز
0
پاسخ : ماراتن نظریه ی اعداد (سطح ممتاز)

برای سوال قبل ثابت کنید تنها چند جمله ای
هست.
سوال بعد:
تمام اعداد صحیح و نامنفی
و عدد اول
را بیابید به طوریکه:

فقط دوستانی که سوال میذارن خیلی سطح سوال رو بالا نبرن و سعی کنین کم کم سوالا رو سخت کنین تا ماراتن دوباره نخوابه!
یک رو میبریم اونور.بعد تجزیه اش میکنیم میشه:


حالا دو حالت داریم:
1)

یعنی
که اگه تو پرانتز دومی بزاریم میشه
که اگه x مثبت باشه y منفی میشه ولی سوال اعداد نا منفی رو میخواد پس تو این حالت جواب نداره.

2)


پس داریم:
که اگه دلتای این رو بنویسیم باید داشته باشیم


که اینم فقط برای
جواب داره چون اون نامساویه برای
جواب نداره برای
هم دستی چک میکنیم که جواب نداره.

که اگه
رو بزاریم در میاد


پس تنها جواب اینه:


---- دو نوشته به هم متصل شده است ----

برای سوال قبل ثابت کنید تنها چند جمله ای
هست.
سوال بعد:
تمام اعداد صحیح و نامنفی
و عدد اول
را بیابید به طوریکه:

فقط دوستانی که سوال میذارن خیلی سطح سوال رو بالا نبرن و سعی کنین کم کم سوالا رو سخت کنین تا ماراتن دوباره نخوابه!
یک رو میبریم اونور.بعد تجزیه اش میکنیم میشه:


حالا دو حالت داریم:
1)

یعنی
که اگه تو پرانتز دومی بزاریم میشه
که اگه x مثبت باشه y منفی میشه ولی سوال اعداد نا منفی رو میخواد پس تو این حالت جواب نداره.

2)


پس داریم:
که اگه دلتای این رو بنویسیم باید داشته باشیم


که اینم فقط برای
جواب داره چون اون نامساویه برای
جواب نداره برای
هم دستی چک میکنیم که جواب نداره.

که اگه
رو بزاریم در میاد


پس تنها جواب اینه:
 

math1998

New Member
ارسال ها
336
لایک ها
224
امتیاز
0
پاسخ : ماراتن نظریه ی اعداد (سطح ممتاز)

یک رو میبریم اونور.بعد تجزیه اش میکنیم میشه:


حالا دو حالت داریم:
1)

یعنی
که اگه تو پرانتز دومی بزاریم میشه
که اگه x مثبت باشه y منفی میشه ولی سوال اعداد نا منفی رو میخواد پس تو این حالت جواب نداره.

2)


پس داریم:
که اگه دلتای این رو بنویسیم باید داشته باشیم


که اینم فقط برای
جواب داره چون اون نامساویه برای
جواب نداره برای
هم دستی چک میکنیم که جواب نداره.

که اگه
رو بزاریم در میاد


پس تنها جواب اینه:


---- دو نوشته به هم متصل شده است ----


یک رو میبریم اونور.بعد تجزیه اش میکنیم میشه:


حالا دو حالت داریم:
1)

یعنی
که اگه تو پرانتز دومی بزاریم میشه
که اگه x مثبت باشه y منفی میشه ولی سوال اعداد نا منفی رو میخواد پس تو این حالت جواب نداره.

2)


پس داریم:
که اگه دلتای این رو بنویسیم باید داشته باشیم


که اینم فقط برای
جواب داره چون اون نامساویه برای
جواب نداره برای
هم دستی چک میکنیم که جواب نداره.

که اگه
رو بزاریم در میاد


پس تنها جواب اینه:
راه حل درسته
هم جوابه اگه سر امتحان ننویسید قطعا یه نمره کم میکنن فقط میخواستم بگم سر امتحان حواستون باشه!!!
 

m-saghaei

New Member
ارسال ها
338
لایک ها
258
امتیاز
0
پاسخ : ماراتن نظریه ی اعداد (سطح ممتاز)

راه حل درسته
هم جوابه اگه سر امتحان ننویسید قطعا یه نمره کم میکنن فقط میخواستم بگم سر امتحان حواستون باشه!!!
بله ممنون
حواسم نبود!!!
 

Dadgarnia

New Member
ارسال ها
1,350
لایک ها
1,127
امتیاز
0
پاسخ : ماراتن نظریه ی اعداد (سطح ممتاز)

یک رو میبریم اونور.بعد تجزیه اش میکنیم میشه:


حالا دو حالت داریم:
1)

یعنی
که اگه تو پرانتز دومی بزاریم میشه
که اگه x مثبت باشه y منفی میشه ولی سوال اعداد نا منفی رو میخواد پس تو این حالت جواب نداره.

2)


پس داریم:
که اگه دلتای این رو بنویسیم باید داشته باشیم


که اینم فقط برای
جواب داره چون اون نامساویه برای
جواب نداره برای
هم دستی چک میکنیم که جواب نداره.

که اگه
رو بزاریم در میاد


پس تنها جواب اینه:


---- دو نوشته به هم متصل شده است ----


یک رو میبریم اونور.بعد تجزیه اش میکنیم میشه:


حالا دو حالت داریم:
1)

یعنی
که اگه تو پرانتز دومی بزاریم میشه
که اگه x مثبت باشه y منفی میشه ولی سوال اعداد نا منفی رو میخواد پس تو این حالت جواب نداره.

2)


پس داریم:
که اگه دلتای این رو بنویسیم باید داشته باشیم


که اینم فقط برای
جواب داره چون اون نامساویه برای
جواب نداره برای
هم دستی چک میکنیم که جواب نداره.

که اگه
رو بزاریم در میاد


پس تنها جواب اینه:
البته فقط يه اشتباه كوچيك كردين
هم جوابه.
سوال بعد رو هم بذارين.
 

m-saghaei

New Member
ارسال ها
338
لایک ها
258
امتیاز
0
پاسخ : ماراتن نظریه ی اعداد (سطح ممتاز)

سوال بعد:
عددی صحیح و نامنفی است.تمام اعداد صحیح و نامنفی
را بیابید که داشته باشیم:
 

Dadgarnia

New Member
ارسال ها
1,350
لایک ها
1,127
امتیاز
0
پاسخ : ماراتن نظریه ی اعداد (سطح ممتاز)

سوال بعد:
عددی صحیح و نامنفی است.تمام اعداد صحیح و نامنفی
را بیابید که داشته باشیم:
اول فرض مي كنيم n=0 باشه در اين حالت جواب ها
و جايگشت هاي اوناست. حالا اگه
باشه داريم:
پس يا همه ي اعداد فرد هستند يا همه ي اعداد زوج. در حالت دوم در اينجا هم دو حالت وجود داره كه در حالت اول جواب
بدست مياد و در حالت دوم داريم
(با توجه به چيز هايي كه بدست آورديم واضحه كه توان ها بايد برابر باشند) و
ها اعداد فردند. اگه اين رابطه رو توي صورت سوال جاگذاري كنيم بدست مياد:

پس كافيه براي اعداد فرد بررسي كنيم. در اين حالت داريم:


پس در كل جواب هاي
و جايگشت هاشون وجود داره.
 
آخرین ویرایش توسط مدیر

math1998

New Member
ارسال ها
336
لایک ها
224
امتیاز
0
پاسخ : ماراتن نظریه ی اعداد (سطح ممتاز)

سوال بعد:
عددی صحیح و نامنفی است.تمام اعداد صحیح و نامنفی
را بیابید که داشته باشیم:
مانده های درجه دو به پیمانه ی 8 اعداد 0و1و4 هستند اگه
باشه میشه چک کرد که یه نزول نامتناهی تشکیل میشه که اعداد
بی نهایت بار بر 2 بخش پذیرن پس یا جواب نداره یه بی نهایت جواب داره که با حل معادلات





البته با احتساب جایگشتاشون!!!


حالا دیگه بقیه جوابا با این جواب اولیه بدست میان!!!

 
آخرین ویرایش توسط مدیر

m-saghaei

New Member
ارسال ها
338
لایک ها
258
امتیاز
0
پاسخ : ماراتن نظریه ی اعداد (سطح ممتاز)

اول فرض مي كنيم n=0 باشه در اين حالت جواب ها
و جايگشت هاي اوناست. حالا اگه
باشه داريم:
پس يا همه ي اعداد فرد هستند يا همه ي اعداد زوج. در حالت دوم در اينجا هم دو حالت وجود داره كه در حالت اول جواب
بدست مياد و در حالت دوم داريم
(با توجه به چيز هايي كه بدست آورديم واضحه كه توان ها بايد برابر باشند) و
ها اعداد فردند. اگه اين رابطه رو توي صورت سوال جاگذاري كنيم بدست مياد:
پس كافيه براي اعداد فرد بررسي كنيم. در اين حالت داريم:


پس در كل جواب هاي
و جايگشت هاشون وجود داره.
اینجوری که شما میگید اگه n=1 باشه باید جواب (8و8و8و4) هم داشته باشیم که اینطور نیست.
 

Dadgarnia

New Member
ارسال ها
1,350
لایک ها
1,127
امتیاز
0
پاسخ : ماراتن نظریه ی اعداد (سطح ممتاز)

مانده های درجه دو به پیمانه ی 8 اعداد 0و1و4 هستند اگه
باشه میشه چک کرد که یه نزول نامتناهی تشکیل میشه که اعداد
بی نهایت بار بر 2 بخش پذیرن پس یا جواب نداره یه بی نهایت جواب داره که با حل معادلات



البته با احتساب جایگشتاشون!!!

حالا دیگه بقیه جوابا با این جواب اولیه بدست میان!!!

البته راه حل شما كامل نيست.
سوال بعد:
تمام اعداد زوج و طبيعي
را بيابيد به طوريكه
كه
ها مقسوم عليه هاي متمايز
هستند.

---- دو نوشته به هم متصل شده است ----

اینجوری که شما میگید اگه n=1 باشه باید جواب (8و8و8و4) هم داشته باشیم که اینطور نیست.
بله حق با شماست اولين جواب بايد اينجوري باشه
 

m-saghaei

New Member
ارسال ها
338
لایک ها
258
امتیاز
0

aras2213

New Member
ارسال ها
216
لایک ها
228
امتیاز
0
پاسخ : ماراتن نظریه ی اعداد (سطح ممتاز)

سوال بعد:
تمام اعداد زوج و طبيعي
را بيابيد به طوريكه
كه
ها مقسوم عليه هاي متمايز
هستند.
1003 , 1620 نسبت به هم اولند.
.حالا جمع مقسوم علیه های 2006 میشه 3240.پس t=1 و n=2006.:7:
 

math1998

New Member
ارسال ها
336
لایک ها
224
امتیاز
0
پاسخ : ماراتن نظریه ی اعداد (سطح ممتاز)

1003 , 1620 نسبت به هم اولند.
.حالا جمع مقسوم علیه های 2006 میشه 3240.پس t=1 و n=2006.:7:
ممکنه بیشتر توضیح بدید؟؟؟
 

aras2213

New Member
ارسال ها
216
لایک ها
228
امتیاز
0
پاسخ : ماراتن نظریه ی اعداد (سطح ممتاز)

ممکنه بیشتر توضیح بدید؟؟؟
بلی!
داریم:
.پس اگه طرفین رو در n ضرب کنیم به
میرسیم.بعد چون سمت چپ طبیعیه سمت راست هم باید طبیعی باشه پس
. حالا چون n زوجه
و ... .

---- دو نوشته به هم متصل شده است ----

سوال بعد:ثابت کنید برای نامتناهی n،
حداقل یک عامل اول بزرگتر از
دارد.(a طبیعی و از 1 بزرگتر است.)

---- دو نوشته به هم متصل شده است ----

آقا من اول که این سوال رو حل کردم فکر کردم آسونه بعد یکم فکر کردم دیدم جوب زدم و یکم سخته.اگه راه آسونی پیدا کردید بگید.این یه سوال دیگه:
ثابت کنید معادله
در اعداد صحیح جواب ندارد.
 
بالا